Archivo de la categoría: Sin clasificar

Conjuntos de Jordan y conjuntos Jordan medibles

Por Ruben Hurtado

$\framebox[5cm][c]{Medida de Jordan}$
$\textcolor{Blue}{Definición}$
Dado $A\subset \mathbb{R}^{n}$ acotado, definimos $\chi_{A}:\mathbb{R}^{n}\rightarrow \mathbb{R}$, la $\textcolor{Red}{función~ característica}$ de A, de la siguiente forma
$$\boxed{\chi_{A}=\left\{\begin{matrix}
1&si&x\in A\\0&si& x\notin A\end{matrix}\right.}$$

Dado $A\subset \mathbb{R}^{n}$ acotado, decimos que A es $\textcolor{Red}{Jordan Medible} si la función característica de A es integrable sobre algún rectángulo R que contenga a A. En este caso decimos que la medida de Jordan de A (que denotaremos J(A)) esta dada por
$$\boxed{J(A)=\int_{R}\chi_{A}}$$
$\textcolor{Green}{Ejemplo:}$
Sean a,b números positivos y $$A=\left\{(x,y)\in \mathbb{R}^{2}| 0\leq x\leq a, 0\leq y \leq \left(\frac{b}{a}x\right)\right\}$$ En este caso A es un triángulo de base a y altura b. Mostraremos que A es Jordan-Medible en $\mathbb{R}^{2}$, y que su medida es $\frac{ab}{2}$
$\textcolor{Green}{Demostración:}$
Tomemos el rectángulo $R=\{(x,y)\in \mathbb{R}^{2}|0\leq x\leq a, 0\leq y \leq b\}=[0,a]\times[0,b]$ el cual contiene a A. De acuerdo con nuestra definición tenemos que mostar que la función característica $\chi_{A}$ es integrable sobre R y que $$\boxed{\int_{R}\chi_{A}=\frac{ab}{2}}$$
Sea $\displaystyle{P_{1}=\left\{\frac{ia}{n}~|~i=1,…,n\right\}}$ y $\displaystyle{P_{2}=\left\{\frac{ib}{n}~|~i=1,…,n\right\}}$ por tanto $P=P_{1}\times P_{2}$ es una partición del rectángulo R, donde cada subrectángulo tiene medida $\displaystyle{\frac{ab}{n^{2}}}$. Por lo que en este caso
$$\overline{S}(\chi_{A},P)=\sum_{R_{i}\cap A\neq \emptyset}=2\cdot \frac{ab}{n^{2}}+3\cdot \frac{ab}{n^{2}}+…+n\cdot \frac{ab}{n^{2}}=\frac{ab}{n^{2}}\left(\frac{n(n+1)}{2}-1\right)=\frac{ab}{2}\left(1+\frac{1}{n}-\frac{1}{n^2}\right)$$
$$\underline{S}(\chi_{A},P)=\sum_{R_{i}\subset A\neq \emptyset}=1\cdot \frac{ab}{n^{2}}+2\cdot \frac{ab}{n^{2}}+…+(n-1)\cdot \frac{ab}{n^{2}}=\frac{ab}{n^{2}}\left(\frac{(n-1)n}{2}\right)=\frac{ab}{2}\left(1-\frac{1}{n}\right)$$
$\therefore$
$$\lim_{n\rightarrow \infty}\overline{S}(\chi_{A},P)=\lim_{n\rightarrow \infty}\underline{S}(\chi_{A},P)=\frac{ab}{2}$$
$$\boxed{J(A)=\int_{R}\chi_{A}=\frac{ab}{2}}$$

$\textcolor{Green}{Ejemplo:}$
Sea A el conjunto $$A=\left\{(x,y)\in \mathbb{R}^{2}~|~ 0\leq x\leq 1, 0\leq y \leq x^{2}\right\}$$ En este caso A la región de base 1 y altura la función $x^{2}$. Mostraremos que A es Jordan-Medible en $\mathbb{R}^{2}$, y que su medida es $\displaystyle{\frac{1}{3}}$
$\textcolor{Green}{Demostración:}$
Tomemos el rectángulo $R=\{(x,y)\in \mathbb{R}^{2}|0\leq x\leq 1, 0\leq y \leq 1\}=[0,1]\times[0,1]$ el cual contiene a A. De acuerdo con nuestra definición tenemos que mostar que la función característica $\chi_{A}$ es integrable sobre R y que $$\boxed{\int_{R}\chi_{A}=\frac{1}{3}}$$
Sea $\displaystyle{P_{1}=\left\{\frac{ia}{n}~|~i=1,…,n\right\}}$ y $\displaystyle{P_{2}=\left\{\frac{ib}{n}~|~i=1,…,n\right\}}$ por tanto $P=P_{1}\times P_{2}$ es una partición del rectángulo R, donde cada subrectángulo tiene medida $\displaystyle{\frac{1}{n^{2}}}$. Por lo que en este caso
$$\overline{S}(\chi_{A},P)=\sum_{R_{i}\cap A\neq \emptyset}=\frac{1}{n}\cdot \frac{1}{n^{2}}+\frac{1}{n}\cdot \frac{2^{2}}{n^{2}}+…+\frac{1}{n}\cdot \frac{n^{2}}{n^{2}}=\frac{1}{n^{3}}\left(1+2^{2}+\cdots+n^{2}\right)=\frac{1}{n^{3}}\left(\frac{n(n+1)(2n+1)}{6}\right)=\frac{1}{6}\left(1+\frac{1}{n}\right)\left(1+\frac{1}{n}\right)$$
$$\underline{S}(\chi_{A},P)=\sum_{R_{i}\subset A\neq \emptyset}=\frac{1}{n}\cdot \frac{1}{n^{2}}+\frac{1}{n}\cdot \frac{2^{2}}{n^{2}}+…+\frac{1}{n}\cdot \frac{(n-1)^{2}}{n^{2}}=\frac{1}{n^{3}}\left(1+2^{2}+\cdots+(n-1)^{2}\right)=\frac{1}{n^{3}}\left(\frac{(n-1)n(2(n-1)+1)}{6}\right)$$
$$=\frac{1}{6}\left(1-\frac{1}{n}\right)\left(2-\frac{1}{n}\right)$$
$\therefore$
$$\lim_{n\rightarrow \infty}\overline{S}(\chi_{A},P)=\lim_{n\rightarrow \infty}\underline{S}(\chi_{A},P)=\frac{1}{3}$$
$$\boxed{J(A)=\int_{R}\chi_{A}=\frac{1}{3}}$$

$\textcolor{Blue}{Teorema}$
Sea $A\subset\mathbb{R}^{n}$ acotado. Las siguientes afirmaciones son equivalentes:
1.- A es Jordan-Medible
2.-Para cada $\epsilon>0$ existen $R_{1},…,R_{k}$ rectángulos tales que:
(a) $Fr(A)\subset R_{1}\cup\cdots\cup R_{k}$
(b) $\displaystyle{\sum_{i=1}^{k}m(R_{i})<\epsilon}$
3.-La $Fr(A)$ es Jordan-Medible y $J(Fr(A))=0$
$\textcolor{Blue}{Demostración:}$
$(1)\Rightarrow (2)$
Sea $\epsilon>0$. Como A es Jordan-Medible, sabemos que $\chi_{A}$ es integrable sobre un rectángulo que contiene $\overline{A}$ por lo que existe una partición P de R tal que
$$\overline{S}(\chi_{A},P)-\underline{S}(\chi_{A},P)<\epsilon$$
en este caso
$$\overline{S}(\chi_{A},P)=\sum_{R_{i}\cap A\neq0}m(R_{i})$$
$$\underline{S}(\chi_{A},P)=\sum_{R_{i}\subset A}m(R_{i})$$
Por lo que
$$\overline{S}(\chi_{A},P)-\underline{S}(\chi_{A},P)=\sum_{R_{i}\cap A\neq \emptyset\atop{R_{i}\cap A^{c}\neq \emptyset}}m(R_{i})$$
Para ver que
$$Fr(A)\subset R_{1}\bigcup\cdots\bigcup R_{k}$$
Como $\underline{A}\subset int(R)$ entonces $Fr(A)\subset int(R)$. Sea $x\in Fr(A)$, si $x\in int(R_{i})$ entonces la condición se cumple
Si $x\notin int(R)$ entonces esta en la frontera de más de uno de los subrectángulos por lo que
(1) Si los subrectángulos estan en $int(A)$ entonces $x\in int(A)$
(2) Si los subrectángulos estan en $int(A^{c})$ entonces $x\in ext(A)$
En ambas situaciones $x\notin Fr(A)$ lo cual es una contradicción.
Por lo tanto $x\in R_{i}$ donde $R_{i}$ cumple la condición pedida.
$(2)\Rightarrow (3)$
Para probar que $Fr(A)$ es Jordan-Medible, se tiene que probar que $\chi_{A}$ es integrable sobre el rectángulo R ya que $Fr(a)\subset R$.
Como existen $R_{1},…,R_{k}$ rectángulos tales que:
\begin{align*}
(a)&Fr(A)\subset R_{1}\bigcap\cdots\bigcap R_{k}\\
(b)&\sum_{i=1}^{k}m(R_{i})<\epsilon
\end{align*}
Entonces
$$\overline{S}(\chi_{Fr(A)},P)=\sum_{R_{i}\cap A\neq\emptyset\atop{R_{i}\cap A^{c}\neq\emptyset}}m(R_{i})<\epsilon$$
Y como
$$\underline{S}(\chi_{Fr(A)},P)\leq \overline{S}(\chi_{Fr(A)},P)<\epsilon$$
entonces
$$0\leq \sup\{\underline{S}(\chi_{Fr(A)},P)\}<\epsilon$$
$$0\leq \inf\{\overline{S}(\chi_{Fr(A)},P)\}<\epsilon$$
entonces
$$\int_{R}\chi_{Fr(a)}=\overline{\int}_{R}\chi_{Fr(A)}=\underline{\int}_{R}\chi_{Fr(A)}=0$$
$(3)\Rightarrow (1)$
Como $Fr(A)$ es Jordan-Medible y $J(Fr(A))=0$, existe una partición P tal que $Fr(A)\cap R_{i}\neq\emptyset$ entonces
$$\overline{S}(\chi_{Fr(A)})=\sum_{R_{i}\cap Fr(A)\neq \emptyset}m(R_{i})<\epsilon$$
Por lo tanto
$$\overline{S}(\chi_{Fr(A)})-\underline{S}(\chi_{Fr(A)})=\sum_{R_{i}\cup A\neq\emptyset\atop{R_{i}\cup A^{c}\neq\emptyset}}m(R_{i})\leq \sum_{R_{i}\cup Fr(A)\neq\emptyset}m(R_{i})<\epsilon$$
por lo tanto $\chi_{A}$ es integrable sobre R y en consecuencia A es un conjunto Jordan-Medible.$~~\blacksquare$

$\textcolor{Blue}{Teorema}$
Sea $f:R\subset\mathbb{R}\rightarrow\mathbb{R}$ acotada sobre el rectángulo R. Si el conjunto de discontinuidades de f en R $(D_{f,R})$ es un conjunto Jordan-Medible y de Medida cero entonces f es integrable sobre R.
$\textcolor{Blue}{Demostración}$
Sea $M>0$ tal que $|f(\widehat{x})|\leq M$ para todo $\widehat{x}\in R$.
Sea $\epsilon>0$. Dado que $J(D_{f,R})=0$ sabemos que existe una partición P de R tal que
$$\overline{S}(\chi_{D_{f,R}},P)<\frac{\epsilon}{4M}$$
Si $R_{1},…,R_{l}$ son los subrectángulos de R inducidos por la partición P que tienen la propiedad de que $R_{i}\cap D_{f,R}\neq\emptyset~~(i=1,2,…,l)$, tenemos entonces que
$$\sum_{i=1}^{l}m(R_{i})<\frac{\epsilon}{4M}$$
Si $R_{\ell+1},…,R_{k}$ son el resto de los subrectángulos de R inducidos por P, sabemos que f es continua en cada uno de ellos por tanto f es integrable sobre cada $R_{i}~~(i=\ell+1,…,k)$. Para cada uno de estos subrectángulos podemos encontrar una partición $P^{(i)}$ tal que
$$\overline{S}(f,P^{(i)})-\underline{S}(f,P^{(i)})<\frac{\epsilon}{2(k-\ell)}$$
Para cada $i=\ell+1,…,k$. Cada una de estas particiones $P^{(i)}$ la extendemos a todo el rectángulo R y junto con la partición inicial P, formamos una nueva partición del rectángulo R a la que llamaremos Q.
si los subrectángulos inducidos por Q los denotamos $R^{‘}_{j,i}$, en donde $(i=1,…,k)$ y $(j=1,..k_{i})$, entonces
$$R_{i}=\bigcup_{j=1}^{k_{i}}R’_{j,i}$$
Observe que para $i=\ell+1,…,k$ los $R’_{j,i}$ $(j=1,…,k_{i})$ son subrectángulo de $R_{i}$ inducidos por alguna partición $Q^{(i)}$ que refina a $P^{(i)}$ de modo tal que
\begin{align*}
\overline{S}(f,Q^{(i)})-\underline{S}(f,Q^{i})&\leq \overline{S}(f,P^{(i)})-\underline{S}(f,P^{i})\\
&<\frac{\epsilon}{2(k-\ell)}
\end{align*}
Tenemos que
\begin{align*}
\overline{S}(f,Q)-\underline{S}(f,Q)&=\sum_{i=1}^{k}\left(\sum_{j=1}^{k}(M’_{j,i}-m’_{j,i})\cdot m(R’_{j,i}\right)\\
&=\sum_{i=1}^{k}\left(\sum_{j=1}^{k}(M’_{j,i}-m’_{j,i})\cdot m(R’_{j,k}\right)+\sum_{i=\ell+1}^{k}\left(\sum_{j=1}^{k}(M’_{j,i}-m’_{j,i})\cdot m(R’_{j,k}\right)\\
&=\sum_{i=1}^{\ell}\left(\sum_{j=1}^{k_{i}}(M’_{j,i}-m’_{j,i})\cdot m(R’_{j,k}\right)+\sum_{i=\ell+1}^{k}\left(\overline{S}(f,Q^{i})-\underline{S}(f,Q^{i})\right)\\
&\leq \sum_{i=1}^{\ell}\left(\sum_{j=1}^{k_{i}}2M\cdot m(R’_{j,i})\right)+\sum_{i=\ell+1}^{k}\left(\overline{S}(f,P^{i})-\underline{S}(f,P^{i})\right)\\
&<2M\cdot \sum_{i=1}^{\ell}\left(\sum_{j=1}^{k_{i}} m(R’_{j,i})\right)+\sum_{i=\ell+1}^{k}\frac{\epsilon}{2(k-\ell)}\\
&=2M\cdot \sum_{i=1}^{\ell}m(R_{i})+\frac{\epsilon}{2}\\
&<2M\cdot \left(\frac{\epsilon}{4M}\right)+\frac{\epsilon}{2}\\
&=\epsilon
\end{align*}
En donde, como era de esperarse, $M’_{j,i}=\sup\{f(\widehat{x})~|~\widehat{x}\in R’_{j,i}\}$ y $m’_{j,i}=\inf\{f(\widehat{x})~|~\widehat{x}\in R’_{j,i}\}$ para $i=1,…,k$ y $j=1,…,k_{i}$.$~~\blacksquare$


Conjuntos de medida cero y contenido cero

Por Ruben Hurtado

Si $f:A\subset\mathbb{R}^{2}\rightarrow \mathbb{R}$. Denotamos por $D_{fA}$ al conjunto de discontinuidades de f en A, es decir
$$D_{fA}=\{x\in A~|~f~es~discontinua~en~x\}$$

$\textcolor{Blue}{Proposición}$: Sea $f:A\subset\mathbb{R}^{2}\rightarrow \mathbb{R}$ integrable sobre R. Entonces $D_{fR}$ tiene interior vacio $int~D_{fR}=\emptyset$
$\textcolor{Blue}{Demostración}$: Si $int~D_{fR}\neq\emptyset$ entonces existe $R’$ tal que $R’\subset int(D_{fR})\subset D_{fR}\subset R$
como f es integrable sobre R y $R’\subset R$ entonces f es integrable sobre $R’$ de modo que existe $x_{0}\in int(R’)$ tal que f es continua en $x_{0}$ lo cual contradice el hecho de que f es discontinua en todo $x\in D_{fR}$.$~~\blacksquare$

Medida Cero y Contenido Cero
$\textcolor{Blue}{Definición}$: Un subconjunto $A\subset\mathbb{R}^{n}$ tiene medida cero si para cada $\epsilon>0$ existe un recubrimiento ${U_{1},U_{2},…}$ de A por rectángulos tales que
$$A\subset \bigcup_{i=1}^{\infty}U_{i}~~y~~\sum_{i=1}^{\infty}v(U_{i})<\epsilon$$
Por ejemplo un conjunto formado por un número finito de puntos claramente tiene medida cero

Si A tiene infinitos puntos que pueden ordenarse formando una sucesión $a_{1},a_{2},…$ entonces A tiene medida cero, pues para cada $\epsilon>0$ se puede elegir $U_{i}$ que sea rectángulo cerrado que contenga $a_{i}$ con $\displaystyle{v(U_{i})<\frac{\epsilon}{2^{i}}}$.
Entonces$$\sum_{i=1}^{\infty}v(U_{i})<\sum_{i=1}^{\infty}\frac{\epsilon}{2^{i}}=\epsilon\sum_{i=1}^{\infty}\frac{1}{2^{i}}=\epsilon\left(\frac{1}{2}+\frac{1}{2^{2}}+\frac{1}{2^{3}}+…\right)=\frac{\epsilon}{2}\left(1+\frac{1}{2}+…\right)=$$
$$\frac{\epsilon}{2}\left(\frac{1}{1-\frac{1}{2}}\right)=\frac{\epsilon}{2}(2)=\epsilon$$

$\textcolor{Green}{Ejemplo}$: $\mathbb{Q}$ es de medida cero.
$\textcolor{Green}{Demostración}$: Como $\mathbb{Q}$ es numerable podemos formar $\displaystyle{\left\{r_{k}\right\}^{\infty}_{1}}$ y dado $\epsilon>0$, sea $$I_{k}=\left(r_{k}-\frac{\epsilon}{2^{k+2}},r_{k}+\frac{\epsilon}{2^{k+2}}\right)$$Por lo que $$\mathbb{Q}\subset \bigcup_{k=1}^{\infty} I_{k}\quad y \quad v(I_{k})=\frac{\epsilon}{2^{k+1}}$$y para la suma de los volumenes se tiene
$$\sum_{k=1}^{\infty}\frac{\epsilon}{2^{k+1}}=\frac{\epsilon}{4}\left(1+\frac{1}{2}+…\right)=\frac{\epsilon}{2}<\epsilon$$
$\therefore$ $\mathbb{Q}$ tiene medida cero.$~~\blacksquare$

$\textcolor{Green}{Ejemplo}$: El conjunto de Cantor
$\textcolor{Green}{Demostración}$: Tenemos que
\begin{align} \mathcal{C}_{0} &=[0,1]\\ \mathcal{C}_{1}&= \left[0,\frac{1}{3}\right] \cup \left[\frac{2}{3},1\right]\\ \mathcal{C}_{2}&= \left[0,\frac{1}{9}\right] \cup \left[\frac{2}{9},\frac{1}{3}\right]\cup \left[\frac{2}{3},\frac{7}{9}\right]\ \cdot\ \cdot\ \cdot\ \mathcal{C}{n}=\left[0,\frac{1}{3^{n}}\right]\cup \cdot \cdot \cdot \cup \left[1-\frac{1}{3^{n}},1\right]
\end{align}
Tenemos que $\displaystyle{\mathcal{C}=\bigcap_{k=0}^{\infty}\mathcal{C}_{k}}$ (Conjunto de Cantor).
Cada $\mathcal{C}_{k}$ es la unión de $2^{k}$ intervalos de longitud $\displaystyle{\frac{1}{3^{k}}}$ si los llamamos $I{1}, I_{2},…,I_{2^{k}}$ entonces
$$\mathcal{C}\subset \mathcal{C}_{k}=\bigcup{i=1}^{2^{k}}I_{i} \quad y \quad \sum_{i=1}^{2^{k}}vol(I_{i})=\left(\frac{2}{3}\right)^{k}$$
por lo que dado $\epsilon>0$ podemos tomar $k$ tal que $\displaystyle{\left(\frac{2}{3}\right)^{k}<\epsilon}$. Entonces $\mathcal{C}$ es de medida cero.$\blacksquare$

$\textcolor{Green}{Ejemplo}$: En $\mathbb{R}^{2}$ consideramos la recta $y=y_{0}$
$\textcolor{Green}{Demostración}: La semirecta derecha $A={(x,y_{0})| x\in \mathbb{R}}$ la cubrimos con $$R_{k}=\left[k,k+1\right]\times \left[y_{0}-\frac{\epsilon}{2^{k+3}},y_{0}+\frac{\epsilon}{2^{k+3}}\right]$$ con $k\in \mathbb{N}\cup {0}$ y $$v(R_{k})=1\cdot \left(\frac{\epsilon}{2^{k+2}}\right)\Rightarrow \sum_{k=1}^{\infty}\frac{\epsilon}{2^{k+2}}=\frac{\epsilon}{2}<\epsilon$$
Mientras que para la semirecta izquierda consideramos $$R_{k}=\left[-k-1,-k\right]\times \left[y_{0}-\frac{\epsilon}{2^{k+3}},y_{0}+\frac{\epsilon}{2^{k+3}}\right]$$
$\therefore$
$$v(R_{k})=1\cdot \left(\frac{\epsilon}{2^{k+2}}\right)\Rightarrow \sum_{k=1}^{\infty}\frac{\epsilon}{2^{k+2}}=\frac{\epsilon}{2}<\epsilon$$
$\therefore$ la recta entera se puede cubrir con una unión numerable de rectángulos cuya suma es menor a $\epsilon$.$\blacksquare$

$\textcolor{Green}{Ejemplo}$: Los intervalos cerrados $[a,b]\subset \mathbb{R}$ con $a<b$ no tienen medida cero.
$\textcolor{Green}{Demostración}$: Supongamos que $[a,b]\subset \bigcup_{n=1}^{\infty}U_{n}$ con $U_{n}$ abierto, como $[a,b]$ es compacto existe una subcubierta finita ${U_{n}}$ con $$\sum_{k=1}^{\infty} v(U_{k})<\epsilon \quad pero \quad \sum_{k=1}^{\infty} v(U_{k})\geq b-a$$ lo cual nos dice que la suma de volumenes no se puede hacer tan pequeña como se desee, por lo que el conjunto dado no es de medida cero.$\blacksquare$

$\textcolor{Blue}{Teorema}$: Si $A=A_{1}\bigcup A_{2}\bigcup …$ y cada $A_{i}$
tiene medida cero, entonces A tiene medida cero.
$\textcolor{Blue}{Demostración}$: Sea $\epsilon>0$. Puesto que cada $A_{i}$ tiene medida cero
$\exists$ un recubrimiento ${U_{i1},U_{i2},…}$ de $A_{i}$ por
rectángulos cerrados tales que la colección de todos los $U_{ij}$
cubren a A y formamos la sucesión numerable $$U_{11},U_{1,2},…$$
$\therefore$
$$\sum_{j=1}^{\infty}v(U_{ij})<\sum_{i=1}^{\infty}\frac{\epsilon}{2^{i}}<\epsilon~\blacksquare$$

$\textcolor{Blue}{Definición}$: Un subconjunto A de $\mathbb{R}^{n}$ tiene contenido cero si para cada $\epsilon>0$ existe un recubrimiento finito ${U_{1},U_{2},…,U_{n}}$ de A por rectángulos tales que $$\sum_{i=1}^{n}v(U_{i})<\epsilon$$

$\textcolor{Blue}{Teorema}$: Si A es compacto y tiene medida cero, entonces A
tiene contenido cero.
$\textcolor{Blue}{Demostración}$: Sea $\epsilon>0$. Puesto que $A$ tiene medida cero,$\exists$ un recubrimiento ${U_{1},U_{2},…}$ de $A$ por rectángulos tales que $\displaystyle{\sum_{i=1}^{\infty}v(U_{i})<\epsilon}$. Dado que A es compacto, un
número finito de ${U_{1},U_{2},…,U_{n}}$ recubren a A y ademas
$\displaystyle{\sum_{i=1}^{n}v(U_{i})<\epsilon}~~\blacksquare$

$\textcolor{Blue}{Teorema}$: Sea $\phi:[a,b]\rightarrow\mathbb{R}$ una función continua. Entonces la gráfica de $\phi$ tiene contenido cero.
$\textcolor{Blue}{Demostración}$: Siendo $\phi$ continua en el compacto $[a,b]$, es
uniformemente continua en dicho intervalo. Es decir dado $\epsilon>0\quad\exists\quad\delta>0$ tal que para $x,y\in[a,b]$ si
$|x-y|<\delta\quad\Rightarrow\quad|f(x)-f(y)|<\frac{\epsilon}{b-a}$.
Sea $n\in\mathbb{N}$ tal que $\frac{b-a}{n}<\delta$ y consideremos
la partición de $[a,b]$ en n partes iguales
$$a=x_{0}<x_{1}<…<x_{n}=b$$ con $x_{i}=a+i\frac{b-a}{n}$ se tiene
entonces que para
$x,y\in[x_{i-1},x_{i}]\Rightarrow|f(x)-f(y)|<\frac{\epsilon}{b-a}$
$\therefore$
$$\sum_{i=1}^{n}(x_{i}-x_{i-1})\frac{\epsilon}{b-a}=\frac{\epsilon}{b-a}\sum_{i=1}^{n}(x_{i}-x_{i-1})=\frac{\epsilon}{b-a}(b-a)=\epsilon~~\blacksquare$$

$\textcolor{Blue}{Teorema}$: Sea R un rectángulo cerrado y $f:R\rightarrow
\mathbb{R}$ una función acotada. Sea $B=\{x\in\mathbb{R}|f\quad
no\quad es \quad continua\quad en\quad x\}$ entonces si B es un
conjunto de contenido cero f es integrable.
$\textcolor{Blue}{Demostración}$: Vamos a dividir los subrectángulos $R_{ij}$ en
$I)R_{ij}\bigcap B\neq\emptyset$ y $II)R_{ij}\bigcap B=\emptyset$
de manera que para los rectángulos I se tiene que B es de contenido cero $\therefore$
$$\sum_{i=1}^{m}\sum_{j=1}^{n}v(R_{ij})<\epsilon$$ Mientras que para
los rectángulos II se tiene que f es continua y por tanto
$$\sum_{i=1}^{m}\sum_{j=1}^{n}M_{ij}-m_{ij}A(R_{ij})<\sum_{i=1}^{m}\sum_{j=1}^{n}\frac{\epsilon}{2A(R)}A(R_{ij})$$
$\therefore$ Dada la partición P de R se tiene que
$$\overline{S}(f,P)-\underline{S}(f,P)=\sum_{i=1}^{m}\sum_{j=1}^{n}M_{ij}-m_{ij}A(R_{ij})+\sum_{i=1}^{m}\sum_{j=1}^{n}v(R_{ij})<\sum_{i=1}^{m}\sum_{j=1}^{n}\frac{\epsilon}{2A(R)}A(R_{ij})+\frac{\epsilon}{2}$$
$$=\frac{\epsilon}{2A(R)}A(R_{ij})\sum_{i=1}^{m}\sum_{j=1}^{n}A(R_{ij})+\frac{\epsilon}{2}=\frac{\epsilon}{2A(R)}A(R)+\frac{\epsilon}{2}=\frac{\epsilon}{2}+\frac{\epsilon}{2}=\epsilon~\blacksquare$$

$\textcolor{Blue}{Teorema}$: Sea f una función definida en un rectángulo R. Si el conjunto S de puntos donde f es discontinua tiene contenido cero, entonces f es integrable sobre R.

$\textcolor{Blue}{Demostración}$: Sea $\epsilon>0$ dado y sea $\textcolor{Red}{R_{1},R_{2},…,R_{k}}$ el conjunto de rectángulos que cubren a S

tal que
$$\sum_{i=1}^{k}A(R_{i})<\epsilon$$
si se colocan sobre cada $\textcolor{Red}{R_{j}}$ un rectángulo $\textcolor{Green}{R’_{j}}$ con el mismo centro pero del doble de dimensiones

se tiene que
$$\sum_{i=1}^{k}A(\textcolor{Green}{R’{i}})=\sum{i=1}^{k}4A(\textcolor{Red}{R_{i}})<4\epsilon$$
Además, entre los rectángulos $\textcolor{Red}{R_{j}}$ habrá un lado más corto. Denotemos su longitud por $2r$

si tomo el conjunto $R’$ el resto de R después que se han eliminado los interiores de los $\textcolor{Red}{R_{j}}$.

Se tiene que sobre $R’$ f es continua y por tanto uniformemente continua por lo tanto
$$|f(p)-f(q)|<\epsilon~si~|p-q|<\delta$$para cualesquiera p,q en $R’$\Sea $P$ una partición de R tal que $|P|<\delta<r$

Vamos a estimar
$$\overline{S}(f,P)-\underline{S}(f,P)$$
Para esto dividimos los rectángulos de la partición P en dos conjuntos
$$\textcolor{Green}{R'{j}}\bigcap \textcolor{Red}{R{j}}\neq\emptyset~~~~\textcolor{Green}{R'{j}}\bigcap \textcolor{Red}{R{j}}=\emptyset$$
se tiene entonces que
$$\overline{S}(f,P)-\underline{S}(f,P)=\sum_{i=1}^{n}\sum_{j=1}^{m}(M_{ij}-m_{ij})A(R_{ij})=$$
$$\left(\sum\sum(M_{ij}-m_{ij})A(R_{ij})\right){\textcolor{Green}{R'{j}}\bigcap \textcolor{Red}{R_{j}}\neq\emptyset}+\left(\sum\sum(M_{ij}-m_{ij})A(R_{ij})\right){\textcolor{Green}{R'{j}}\bigcap \textcolor{Red}{R_{j}}=\emptyset}<4M\epsilon+\epsilon A=\epsilon(4M+A)$$
donde $M=\sup{f(x)}$ sobre R.$~\blacksquare$

Propiedades de las integrales (Criterio de integrabilidad)

Por Ruben Hurtado

Sumas superiores y sumas inferiores

Denotaremos por $\underline{S}(f)$ al conjunto de todas las sumas inferiores de una función f (Definida sobre el rectángulo R) y como $\overline{S}(f)$ al conjunto de todas las sumas inferiores es decir
$$\underline{S}(f)=\{\underline{S}(f,P)~|~P~\in~P_{R}\}$$
$$\overline{S}(f)=\{\overline{S}(f,P)~|~P~\in~P_{R}\}$$
Definición: Al supremo del conjunto $\underline{S}(f)$ lo
llamamos integral inferior de f sobre R y se puede denotar $\displaystyle{\underline{\int}(f)}$. Y al ínfimo del conjunto $\overline{S}(f)$ lo llamamos integral superior de f sobre R y se puede denotar $\displaystyle{\overline{\int}(f)}$.
Definición: Sea $f:R\subset\mathbb{R}^{n}\rightarrow\mathbb{R}$ acotada sobre el
rectángulo R. Decimos que f es integrable según Riemann sobre R si se tiene que la integral inferior y la integral superior de f sobre R son iguales. Es decir $$\underline{\int}R_{f}=\overline{\int}R_{f}$$ En este caso, a
este número lo llamaremos la integral de f y lo denotarremos por
$$\int\int_{R}f$$
Lema: Para cada $\epsilon>0$ existe una partición P de R tal que
$$0\leq \underline{\int}{R}f-\underline{S}(f,p)<\epsilon~~y~~0\leq \overline{S}(f,p)-\overline{\int}{R}f<\epsilon$$

Demostración:

Como
$$\underline{\int}{R}(f)=\sup{\underline{S}(f)}~ \exists~ P_{1}\in~P_{R}~tal~que$$
$$\underline{\int}{R}(f)-\sup{\underline{S}(f)}<\epsilon$$ $$\overline{\int}{R}(f)=\inf{\overline{S}(f)}~\exists~ P_{2}\in~P_{R}~ tal~que$$
$$\inf{\overline{S}(f)}-\underline{\int}{R}(f)<\epsilon$$ Sea $P=P{1}\bigcup P_{2}$ se tiene entonces que
$$\overline{S}(f,P)\leq \overline{S}(f,P_{2})-\underline{S}(f,P_{1})\leq \underline{S}(f,P)$$
por lo tanto
$$0\leq \underline{\int}{R}f-\underline{S}(f,P)\leq \underline{\int}{R}f-\underline{S}(f,P_{1})<\epsilon$$
$$0\leq \overline{S}(f,P)-\overline{\int}{R}f\leq \underline{S}(f,P{2})-\underline{\int}_{R}f<\epsilon~~\blacksquare$$
Teorema: Sea $f:R\subset\mathbb{R}^{n}\rightarrow\mathbb{R}$ acotada sobre el rectángulo R. Se tiene que f es integrable sobre R si y solo si para cada $\epsilon>0$ existe una P partición de R tal que $$\overline{S}(f,P)-\underline{S}(f,P)<\epsilon$$
Demostración: Sea $\epsilon>0$ como f es integrable $\underline{\int}{R}f=\overline{\int}{R}f=I$ y por las propiedades del supremo sabemos que para $\displaystyle{\frac{\epsilon}{2}>0}$ $\exists$ una $P\prime$ partición de R tal que
$$I-\frac{\epsilon}{2}\leq\underline{S}(f,P\prime)\leq I.$$ Por otra parte de las propiedades del ínfimo sabemos que $\exists$ una $Q\prime$ partición de R tal que
$$I\leq\overline{S}(f,Q\prime)\leq I+\frac{\epsilon}{2}.$$ Si hacemos $P=P\prime\bigcup Q\prime$ tenemos que $$\underline{S}(f,P\prime)\leq\underline{S}(f,P)\leq\overline{S}(f,P)\leq\overline{S}(f,P\prime)$$
$\therefore$
$$I-\frac{\epsilon}{2}\leq\underline{S}(f,P)\leq\overline{S}(f,P)\leq I+\frac{\epsilon}{2}$$
$\therefore$
$$\overline{S}(f,P)-\underline{S}(f,P)\leq I+\frac{\epsilon}{2}-(I-\frac{\epsilon}{2})=\epsilon.$$
$\therefore$ f es integrable.

$(\Leftarrow)$ Tenemos que probar que la función es integrable es decir $\displaystyle{\underline{\int}{R}f=\overline{\int}{R}f}$ o equivalentemente
$\displaystyle{\underline{\int}{R}f-\overline{\int}{R}f=0}$ sabemos que
$$\underline{\int}{R}f\leq\overline{\int}{R}f\Rightarrow
0\leq\overline{\int}{R}f-\underline{\int}{R}f.$$ Sea $\epsilon>0$
por hipótesis, existe P partición de R tal que
$\overline{S}(f,P)-\underline{S}(f,P)<\epsilon$ por otra parte se
tiene que:
$$\underline{S}(f,P)\leq\underline{\int}{R}f\leq\overline{\int}{R}f\leq\overline{S}(f,P)$$
$\therefore$
$$0\leq\overline{\int}{R}f-\underline{\int}{R}f\leq\overline{S}(f,P)-\underline{S}(f,P)<\epsilon$$
$\therefore$
$$\overline{\int}{R}f=\underline{\int}{R}f$$ $\therefore$ f es
integrable. $\blacksquare$
Teorema: Si una función f es continua en un rectángulo $Q=[a,b]\times[c,d]$ entonces
f es integrable en Q.

Demostración: Tenemos que
$$\overline{S}(f,P)-\underline{S}(f,P)=\sum_{i=1}^{m}\sum_{j=1}^{n}M_{ij}\Delta (R_{ij})-\sum_{i=1}^{m}\sum_{j=1}^{n}m_{ij}\Delta (R_{ij})=\sum_{i=1}^{m}\sum_{j=1}^{n}(M_{ij}-m_{ij})\Delta (R_{ij})$$
$$\sum_{i=1}^{m}\sum_{j=1}^{n}\frac{\epsilon}{A(R)}\Delta (R_{ij})=\frac{\epsilon}{A(R)}\sum_{i=1}^{m}\sum_{j=1}^{n}\Delta (R_{ij})=\frac{\epsilon}{A(R)}A(R)$$
La desigualdad se justifica de la siguiente forma: Como f es continua en un conjunto cerrado y acotado entonces f es uniformemente continua $\therefore$ para $$\overline{x_{ij}},\overline{y_{ij}}\in R_{ij}~con~~~|x_{ij}-y_{ij}|<\delta\Rightarrow
|f(x_{ij})-f(y_{ij})|<\frac{\epsilon}{A(R)}~~\blacksquare$$
Ejemplo: Si f es una función integrable sobre R y $R_{k}\subset R$ entonces f es integrable sobre $R_{k}$
Demostracion: Como f es integrable sobre R existe una partición P de R tal que
$$\overline{S}(f,P)-\underline{S}(f,P)<\epsilon$$
Sean $P_{1}$ los $R_{i,j}\in P$ tal que $R_{ij}\subset R_{k}$ y sea $P_{2}$ los $R_{ij}$ restantes, tenemos entonces que
$$\overline{S}(f,P)=\overline{S}(f,P_{1})+\overline{S}(f,P_{2})$$
$$\underline{S}(f,P)=\underline{S}(f,P_{1})+\underline{S}(f,P_{2})$$
por lo tanto
$$\overline{S}(f,P_{1})-\underline{S}(f,P_{1})+\overline{S}(f,P_{2})-\underline{S}(f,P_{2})=\overline{S}(f,P)-\underline{S}(f,P)<\epsilon$$
como cada término de la suma es positivo se tiene que
$$\overline{S}(f,P_{1})-\underline{S}(f,P_{1})<\epsilon$$
y tenemos una partición de $R_{k}\subset R$ donde f es integrable.$~~\blacksquare$
Teorema: Sea $f:R\subset\mathbb{R}^{2}\rightarrow \mathbb{R}$ integrable sobre R. Entonces existe $x_{0}\in~int (R)$ tal que f es continua en $x_{0}$
Demostración: Como f es integrable sobre R existe una partición P de R tal que
$$\overline{S}(f,P)-\underline{S}(f,P)=\sum_{i=1}^{n}\sum_{j=1}^{m}(M_{ij}-m_{ij})A(R)<A(R)$$ como los sumandos son términos no negativos, existen subíndices $ij_{1}$ tal que $M_{ij_{1}}-m_{ij_{1}}<1$ pues de lo contrario si
$$M_{ij}-m_{ij}\geq1~\forall i,j~~\Rightarrow~(M_{ij}-m_{ij})A(R_{ij})\geq A(R_{ij})~\Rightarrow~\sum_{i=1}^{n}\sum_{j=1}^{m}(M_{ij}-m_{ij})A(R_{ij})\geq \sum_{i=1}^{n}\sum_{j=1}^{m}(R_{}ij)=A(R)$$ lo cual contradice nuestra suposición.
Ahora bien sea $R_{ij_{1}}$ el subrectángulo de la partición P que cumple $M_{ij_{1}}-m_{ij_{1}}<1$ donde $R_{ij_{1}}\subset R$.
Como f es integrable sobre R y $R_{ij_{1}}\subset R$ entonces f es integrable en $R_{ij_{1}}$, existe entonces una partición $P_{1}$ de $R_{ij_{1}}$
tal que
$$\overline{S}(f,P_{1})-\underline{S}(f,P_{1})=\sum_{i=1}^{n_{1}}\sum_{j=1}^{m_{1}}(M_{ij_{1}}-m_{ij_{1}})A(R_{ij_{1}})<\frac{A(R_{ij_{1}})}{2}$$Podemos asegurar que existe un subrectángulo $R_{ij_{2}}$ inducido por $P_{1}$ con la propiedad $\displaystyle{M_{ij_{2}}-m_{ij_{2}}<\frac{1}{2}}$ donde
$$M_{ij_{2}}=\sup\{f(x)~|~x\in R_{ij_{2}}\}~~~m_{ij_{2}}=\inf\{f(x)~|~x\in R_{ij_{2}}\}$$ y además $R_{ij_{2}}\subset R_{ij_{1}}$.

Siguiendo este procedimiento, obtenemos una sucesión de rectángulos ${R_{ij_{k}}}$ anidados en $\mathbb{R}^{2}$ con la propiedad
$$M_{ij_{k}}-m_{ij_{k}}<\frac{1}{k}~donde~M_{ij_{k}}=\sup\{f(x)~|~x\in R_{ij_{k}}\}~m_{ij_{k}}=\inf\{f(x)~|~x\in R_{ij_{k}}\}~R_{ij_{k+1}}\subset R_{ij_{k}}$$

Sabemos que
$$\bigcap_{k=1}^{\infty}R_{ij_{k}}\neq\emptyset$$
Ahora, si $$x_{0}\in\bigcap_{k=1}^{\infty}R_{ij_{k}}$$ vamos a comprobar que f es continua en $x_{0}$. Sea $\epsilon>0$ y $N\in\mathbb{N}$ tal que $\displaystyle{\frac{1}{N}<\epsilon}$. Como $x_{0}\in R_{ij_{N+1}}\subset R_{ij_{N}}$ existe un $\delta>0$ tal que
$$B_{\delta}(x_{0})\subset R_{ij_{N}}$$ de tal forma que
$$m_{ij_{N}}\leq f(x)\leq M_{ij_{N}}~\forall x\in B_{\delta}(x_{0})$$ como $\displaystyle{M_{ij_{N}}-m_{ij_{N}}<\frac{1}{N}}$ se tiene que
$$|f(x)-f(x_{0})|<\frac{1}{N}<\epsilon~\forall x\in B_{\delta}(x_{0})$$por lo tanto f es continua en $x_{0}$.$~~\blacksquare$
Ejercicio:Sea $f:R\subset\mathbb{R}^{2}\rightarrow \mathbb{R}$ integrable sobre R tal que $f(x)\geq0$ $\forall~x\in~R$. Si f es continua en $x_{0}\in R$ y $f(x_{0})>0$ entonces $$\int_{R}f>0$$
Solución: Si f es continua en $x_{0}\in R$ y $f(x_{0})>0$ $\exists~\delta>0$ tal que
$$|f(x)-f(x_{0})|<\frac{f(x_{0})}{2}$$ por lo tanto $$\frac{f(x_{0})}{2}0$$
por lo tanto
$$0<\underline{S}(f,P)\leq\int_{R}f~~\blacksquare$$


Integral de una función de dos variables como volumen de una superficie

Por Ruben Hurtado

Dada una función de dos variables que está definida sobre el rectángulo cerrado
$$R=[a,b]\times[c,d]={(x,y)\in\mathbb{R}^{2}\mid a\leq x\leq b,
c\leq y \leq d}$$

suponiendo que $f(x,y)\geq 0$. La gráfica de f es
una superfície con ecuación $z=f(x,y)$. Sea S el sólido que esta
encima de R y debajo de la gráfica de f, es decir
$$S={(x,y,z)\in \mathbb{R}^{3}\mid 0\leq z\leq f(x,y),(x,y)\in R}$$

El volumen en esta caso de S es una aproximación al volumen por debajo de la superficie. Ahora bien si dividimos el rectángulo R en subrectángulos. Para el intervalo [a,b] tenemos m subintervalos $[x_{i-1},x_{i}]$ con una longitud de $\displaystyle{\Delta_{x}=\frac{b-a}{m}}$. Para el intervalo [c,d] tenemos n
subintervalos $[y_{j-1},y_{j}]$ con una longitud de $\displaystyle{\Delta_{y}=\frac{d-c}{n}}$. Al trazar rectas paralelas a los ejes coordenados a través de los puntos extremos de las particiones formamos los subrectángulos
$$R_{ij}=[x_{i-1},x_{i}]\times[y_{j-1},y_{j}]={(x,y)\in\mathbb{R}^{2}\mid x_{i-1}\leq x\leq
x_{i},y_{j-1}\leq y \leq y_{j}}$$ cada uno con un área igual a $\Delta_{A}=\Delta_{x}\Delta_{y}$. Si elegimos un punto muestra $$(x^{*}_{i},y ^{*} _{j})\in R_{ij}$$, entonces podemos aproximar la parte de S que esta encima de cada $R_{ij}$ mediante una caja rectangular delgada con base $R_{ij}$ y altura $$f( x^{*}_{i},y ^{*} _{j} )$$

El volumen de la caja es el producto del área de su base por su
altura, por lo tanto una aproximación al volumen de S es:

$$ V\approx\sum\limits_{i=1}^{m} \sum\limits_{j=1}^{n} f( x^{*}_{i},y ^{*} _{j}) \Delta_{x}\Delta_{y}$$

Con un desarrollo análogo para un conjunto S el sólido que esta
encima de R y encima de la gráfica de f, es decir
$$S={(x,y,z)\in \mathbb{R}^{3}\mid 0\leq f(x,y)\leq z~|~(x,y)\in R}$$

Obtenemos también una aproximación al volumen que se encuentra por debajo de la superficie.
Si consideramos ahora $M_{ij}=sup {f(x_{i},y_{j})}$ y
$m_{ij}=\acute{i}nf{ f(x_{i},y_{j})}$ con $(x_{i},y_{j})\in
R_{ij}$ podemos deducir que
$$\sum_{i=1}^{m}\sum_{j=1}^{n}m_{ij}\Delta R_{ij}\leq V(S)\leq \sum_{i=1}^{m}\sum_{j=1}^{n}M_{ij}\Delta R_{ij}$$
Definición.-Sean f una función (de valores reales) definida y
acotada sobre un rectángulo R contenido en $\mathbb{R}^{n}$ y P una
partición de R. Si $R_{1},R_{2},…,R_{k}$ son los subrectángulos de
R inducidos por la partición P, definimos la suma inferior de f
correspondiente a la partición P denotada por $\underline{S}(f,p)$
como $$\underline{S}(f,p)=\sum_{i=1}^{m}\sum_{j=1}^{n}m_{ij}\Delta
R_{ij}$$ Analogamente definimos la suma superior de f
correspondiente a la partición P denotada por $\overline{S}(f,p)$
como
$$\overline{S}(f,p)=\sum_{i=1}^{m}\sum_{j=1}^{n}M_{ij}\Delta
R_{ij}$$

Estas sumas tienen una serie de propiedades
Proposición 1: Si P es cualquier partición de R, entonces $$\underline{S}(f,p)\leq\overline{S}(f,p)$$
Demostración: Como $m_{ij}=\acute{i}nf{ f(x_{i},y_{j})}$ y $M_{ij}=sup
{f(x_{i},y_{j})}$ se tiene que $$m_{ij}\leq M_{ij}\Rightarrow
m_{ij}\Delta R_{ij}\leq M_{ij}\Delta
R_{ij}\Rightarrow\sum_{i=1}^{m}\sum_{j=1}^{n}m_{ij}\Delta R_{ij}\leq
\sum_{i=1}^{m}\sum_{j=1}^{n}M_{ij}\Delta R_{ij}\Rightarrow
\underline{S}(f,p)\leq \overline{S}(f,p)~\blacksquare$$
Proposición 2: Si $P,Q\in P_{R}$. Si Q refina a P entonces
$$\underline{S}(f,P)\leq\underline{S}(f,Q)\quad y\quad \overline{S}(f,Q)\leq\overline{S}(f,P)$$
Demostración: Sean $R_{1},…,R_{k}$ los subrectángulos inducidos por
P y $R_{1}^{i},…,R_{k}^{i}$ los subrectángulos inducidos por Q.
Dado que cada $R_{j}^{i}$ está contenido en $R_{i}$, tenemos que
${f(\overline{x})\mid \overline{x}\in R_{j}^{i}}\subset
{f(\overline{x})\mid \overline{x}\in R_{i}}$ y por lo tanto
$inf{f(\overline{x})\mid \overline{x}\in R_{i}}\leq
inf{f(\overline{x})\mid \overline{x}\in R_{i}^{j}}$ y
$sup{f(\overline{x})\mid \overline{x}\in R_{j}^{i}}\leq
sup{f(\overline{x})\mid \overline{x}\in R_{i}}$ $\therefore$
$$inf{f(\overline{x})\mid \overline{x}\in R_{i}}\times\Delta R_{ij}\leq
inf{f(\overline{x})\mid \overline{x}\in R_{i}^{j}}\times\Delta
R_{ij}$$
$$sup{f(\overline{x})\mid \overline{x}\in R_{j}^{i}}\times\Delta R_{ij}\leq
sup{f(\overline{x})\mid \overline{x}\in R_{i}}\times\Delta
R_{ij}$$ Si ahora sumamos ambas desigualdades corriendo los índices
i,j se tiene que
$$\sum_{i=1}^{m}\sum_{j=1}^{n}inf{f(\overline{x})\mid \overline{x}\in R_{i}}\leq\sum_{i=1}^{m}\sum_{j=1}^{n}inf{f(\overline{x})\mid \overline{x}\in R_{i}^{j}}$$
$$\sum_{i=1}^{m}\sum_{j=1}^{n}sup{f(\overline{x})\mid \overline{x}\in R_{j}^{i}}\leq\sum_{i=1}^{m}\sum_{j=1}^{n}sup{f(\overline{x})\mid \overline{x}\in R_{i}}$$
Recordando la definición de suma inferior y suma superior se tiene
que$$\underline{S}(f,P)\leq\underline{S}(f,Q)\quad y\quad
\overline{S}(f,Q)\leq\overline{S}(f,P) ~\blacksquare $$
Proposición 3: Si P y Q son cualesquiera dos particiones del
rectángulo R entonces se cumple $$\underline{S}(f,P)\leq
\overline{S}(f,Q)$$
Demostración: Consideremos la partición $P\bigcup Q$. Esta partición
refina tanto a P como a Q de tal forma que, por la proposición 2 se
tiene $$\underline{S}(f,P)\leq\underline{S}(f,P\bigcup Q)$$ y
también
$$\overline{S}(f,P\bigcup Q)\leq\overline{S}(f,Q)$$ Como $$\underline{S}(f,P\bigcup Q)\leq\overline{S}(f,P\bigcup
Q)$$ por la proposición 1, se tiene que
$$\underline{S}(f,P)\leq\overline{S}(f,Q) ~\blacksquare $$
Ejemplo: Estimar el volúmen de la superfície delimitada por el
rectángulo $[0,\pi]\times[0,\pi]$ y la superfície
$f(x,y)=\sin(x+y)$

Vamos a subdividir el rectángulo $[0,\pi]\times[0,\pi]$ como se
muestra en la figura

Tenemos por tanto que $$V\approx
\sum_{i=1}^{2}\sum_{j=1}^{2}f(x_{i},y_{j})\triangle
A=f(0,0)\triangle A+f(0,\frac{\pi}{2})\triangle
A+f(\frac{\pi}{2},0)\triangle
A+f(\frac{\pi}{2},\frac{\pi}{2})\triangle A$$
$$=0\times\left(\frac{\pi^{2}}{2}\right)+1\times\left(\frac{\pi^{2}}{2}\right)+1\times\left(\frac{\pi^{2}}{2}\right)+0\times\left(\frac{\pi^{2}}{2}\right)=\frac{\pi^{2}}{2}\approx4.935$$
Definición: Al supremo del conjunto $\underline{S}(f)$ lo
llamamos integral inferior de f sobre R y se puede denotar

$$\underline{\int}R_{f}$$
Y al ínfimo del conjunto $\overline{S}(f)$ lo llamamos integral superior de f sobre R y podemos denotar

$$\overline{\int}R_{f}$$

Definición: Sea
$f:R\subset\mathbb{R}^{n}\rightarrow\mathbb{R}$ acotada sobre el
rectángulo R. Decimos que f es integrable según Riemann sobre R si
se tiene que la integral inferior y la integral superior de f sobre
R son iguales. Es decir

$$\underline{\int}R_{f}=\overline{\int}R_{f}$$

En este caso, a este número lo llamaremos la integral de f y lo denotaremos por
$\displaystyle{\int\int_{R_{f}}}$

Ejemplo: Calcular $\displaystyle{\underline{\int}R_{f}}~~y~~\displaystyle{\overline{\int}R_{f}}$ para $f(x,y)=x+4y$ en el rectángulo $R=[0,2]\times[0,1]$

Solución: Tenemos que para $[0,2]$
consideramos una partición $P={x_{0},x_{1},…,x_{n}}$ con
longitud $\displaystyle{\frac{2-0}{2n}=\frac{1}{n}}$

de esta manera se tiene que $\displaystyle{x_{i}=\frac{i}{n}}$ y
$\displaystyle{x_{i-1}=\frac{i-1}{n}}$. Mientras que para $[0,1]$ consideramos una
partición $P={y_{0},y_{1},…,y_{n}}$ con longitud
$\displaystyle{\frac{1-0}{n}=\frac{1}{n}}$ de esta manera se tiene que
$\displaystyle{y_{j}=\frac{j}{n}}$ y $\displaystyle{y_{j-1}=\frac{j-1}{n}}$.

$\therefore$ Para todo rectángulo $R_{ij}$,
$M_{ij}=sup{f(x_{i,j})|x_{ij}\in
[x_{i-1},x:{i}]\times[y_{j-1},y_{j}]}=x_{i}+4y_{j}$ y
$m_{ij}=sup{f(x_{i,j})|x_{ij}\in
[x_{i-1},x:{i}]\times[y_{j-1},y_{j}]}=x_{i-1}+4y_{j-1}$
$\therefore$
$$\underline{S}(f,P)=\sum_{i=1}^{2n}\sum_{j=1}^{n}\left(x_{i-1}+4y_{j-1}\right)\left(\frac{1}{n}\right)\left(\frac{1}{n}\right)=\sum_{i=1}^{2n}\sum_{j=1}^{n}\left(\frac{i-1}{n}+4\frac{j-1}{n}\right)\left(\frac{1}{n}\right)\left(\frac{1}{n}\right)$$
$$=\left(\frac{1}{n^{2}}\right)\sum_{i=1}^{2n}\sum_{j=1}^{n}\left(\frac{i-1}{n}+4\frac{j-1}{n}\right)=\left(\frac{1}{n^{3}}\right)\sum_{i=1}^{2n}\sum_{j=1}^{n}i+4j-5=\left(\frac{1}{n^{3}}\right)\sum_{i=1}^{2n}n(i-5)+4\left(n\left(\frac{n+1}{2}\right)\right)$$
$$=\left(\frac{1}{n^{2}}\right)\sum_{i=1}^{2n}(i-5)+2(n+1)=\left(\frac{1}{n^{2}}\right)\sum_{i=1}^{2n}i+2n-3=\left(\frac{1}{n^{2}}\right)\left(2n(2n-3)+\frac{2n(2n+1)}{2})\right)=$$
$$\left(\frac{1}{n}\right)\left(2(2n-3)+2n+1\right)=\left(\frac{1}{n}\right)(4n-6+2n+1)=\left(\frac{1}{n}\right)(6n-5)=6-\frac{5}{n}$$
$\therefore$
$$\underline{\int}R_{f}=\sup\underline{S}(f,P)=\lim_{n\rightarrow\infty}\underline{S}(f,P)= \lim_{n\rightarrow\infty} 6-\frac{5}{n}=6$$

Ahora bien para $\displaystyle{\overline{S}R_{f}}$

$\therefore$
$$\overline{S}(f,P)=\sum_{i=1}^{2n}\sum_{j=1}^{n}\left(x_{i}+4y_{j}\right)\left(\frac{1}{n}\right)\left(\frac{1}{n}\right)=\sum_{i=1}^{2n}\sum_{j=1}^{n}\left(\frac{i}{n}+4\frac{j}{n}\right)\left(\frac{1}{n}\right)\left(\frac{1}{n}\right)$$
$$=\left(\frac{1}{n^{3}}\right)\sum_{i=1}^{2n}\sum_{j=1}^{n}\left(i+4j\right)=\left(\frac{1}{n^{3}}\right)\sum_{i=1}^{2n}ni+4\left(\frac{n(n+1)}{2}\right)=\left(\frac{1}{n^{3}}\right)\left(n\left(\frac{2n(2n+1)}{2}\right)+2n\left(\frac{4n(n+1)}{2}\right)\right)$$
$$=\left(\frac{1}{ n^{3}}\right)(2n^{3}+n^{2}+4n^{3}+4n^{2})=2+\frac{1}{n}+4+\frac{4}{n}=6+\frac{5}{n}$$

$\therefore$
$$\overline{\int}R_{f}=\sup\overline{S}(f,P)=\lim_{n\rightarrow\infty}\overline{S}(f,P)= \lim_{n\rightarrow\infty} 6+\frac{5}{n}=6$$

Volumen

Por Ruben Hurtado

Volumen

Cuando definimos volumen aceptaremos el hecho de que si se trata de
un cubo de lado a entonces $V(cubo)=a^{3}$ y si se trata de un
cilíndro circular recto de radio r y altura h entonces
$V(cil\acute{i}ndro)=\pi r^{2}h$


Ejemplo.- Volumen de un cono de altura a.


Para esto, dividamos la altura en n partes iguales, cada una de longitud $\displaystyle{\frac{a}{n}}$. Construyamos los n cilindros de altura $\displaystyle{\frac{a}{n}}$ y radio $r_{k}$, k=1,…,n donde $\displaystyle{r_{k}=k\frac{r}{n}}$.

Entonces el volumen del k-ésimo cilindro es
$$V_{k}=\pi r_{k}^{2}a_{k}=\pi \left(k\frac{r}{n}\right)^{2}\left(\frac{a}{n}\right)=\frac{\pi ar^{2}k^{2}}{n^{3}}$$
Por lo tanto el volumen del cono es
$$V\approx \sum_{k=1}^{n}\frac{\pi ar^{2}k^{2}}{n^{3}}=\frac{\pi
ar^{2}}{n^{3}}\sum_{k=1}^{n}k^{2}=\frac{\pi
ar^{2}k^{2}}{n^{3}}\frac{n(n+1)(2n+1)}{6}=\frac{\pi
ar^{2}}{6}\left(1+\frac{1}{n}\right)\left(2+\frac{1}{n}\right)
$$
En consecuencia
$$V=\lim_{n\rightarrow \infty}\frac{\pi
ar^{2}}{6}\left(1+\frac{1}{n}\right)\left(2+\frac{1}{n}\right)=\frac{1}{3}\pi a r^{2}$$

Ejemplo. Volumen de una esfera


Para esto fijémonos en la mitad de la esfera

El radio del k-ésimo cilindro es
$$r_{k}=\sqrt{r^{2}-\left(k\frac{r}{n}\right)^{2}}$$
es decir
$$r_{k}^{2}=r^{2}-\left(k\frac{r}{n}\right)^{2}$$
entonces el volumen del k-ésimo cilindro es
$$V=\pi r_{k}^{2}\frac{r}{n}=\pi
\left(r^{2}-\left(k\frac{r}{n}\right)^{2}\right)\frac{r}{n}=\pi
r^{2}\left(1-\frac{k^{2}}{n^{2}}\right)\frac{r}{n}=\pi
r^{3}\left(1-\frac{k^{2}}{n^{2}}\right)\frac{1}{n}$$
Es la mitad de la esfera, por lo que
$$V\approx 2\sum_{k=1}^{n} \pi
r^{3}\left(1-\frac{k^{2}}{n^{2}}\right)\frac{1}{n}=2 \pi
r^{3}\left(\frac{1}{n}\sum_{k=1}^{n}1-\frac{1}{n^{3}}\sum_{k=1}^{n}k^{2}\right)$$
$$=2 \pi r^{3}\left(1-\frac{1}{6}\left(1+\frac{1}{n}\right)\left(2+\frac{1}{n}\right)\right)
$$
Por lo tanto
$$V=\lim_{n\rightarrow\infty}2 \pi
r^{3}\left(1-\frac{1}{6}\left(1+\frac{1}{n}\right)\left(2+\frac{1}{n}\right)\right)=\frac{4}{3}\pi r^{3}$$

Ejemplo.- ¿Cual es el volumen del sólido que esta acotado superiormente por un plano e inferiormente por un cilindro?

Para resolver esto, dividimos en triángulos rectángulos

Tenemos que según la figura
$$\left(\frac{l_{k}}{2}\right)^{2}+\left(\frac{kr}{n}\right)^{2}=r^{2}$$
por lo tanto
$$l_{k}=2\sqrt{r^{2}-\left(k\frac{r}{n}\right)^{2}},~~\overline{PQ}=k\frac{a}{n}$$

se tiene entonces que
$$V_{k}=\left(2\sqrt{r^{2}-\left(k\frac{r}{n}\right)^{2}}\right)\left(\frac{ka}{n}\right)\left(\frac{r}{n}\right)$$
$$V\approx
2r^{2}a\sum_{k=1}^{n}\left(\frac{k}{n}\right)\sqrt{1-\left(\frac{k}{n}\right)^{2}}\left(\frac{1}{n}\right)$$

$$V=2r^{2}a\lim_{n\rightarrow
\infty}\sum_{k=1}^{n}\left(\frac{k}{n}\right)\sqrt{1-\left(\frac{k}{n}\right)^{2}}\left(\frac{1}{n}\right)=2r^{2}a\int_{0}^{1}x\sqrt{1-x^{2}}dx=\frac{2r^{2}a}{3}$$